• Không có kết quả nào được tìm thấy

Đoàn Ngọc Dũng -

N/A
N/A
Protected

Academic year: 2022

Chia sẻ "Đoàn Ngọc Dũng -"

Copied!
18
0
0

Loading.... (view fulltext now)

Văn bản

(1)

TOÁN TẬP HỢP NÂNG CAO

GVBM : ĐOÀN NGỌC DŨNG BÀI 1 : Có thể nói gì về các tập A và B trong mỗi trường hợp sau:

1) A  B = A. 2) A  B = B. 3) A  B = A. 4) A  B = B.

5) A \ B = A. 6) B \ A = B. 7) A \ B = B \ A. 8) X  A và X  B

BÀI 2 : Xét các tập hợp A = (2m – 1 ; 2m + 3) và B = (1 ; 1), với m là số thực. Tìm m để :

1) A  B 2) B  A 3) A  B = 

ĐS : 1) m   ; 2) 1  m  0 ; 3) m  2 hoặc m  1.

BÀI 3 : Cho hai đoạn A = [a ; a + 2] và B =[–1 ; 1]. Các số a, b cần thỏa mãn điều kiện gì để A  B  .

ĐS : – 3  a  1

BÀI 4 : Cho hai đoạn A = [a ; a + 2] và B =[b ; b + 1]. Các số a, b cần thỏa mãn điều kiện gì để A  B  .

ĐS : b – 2  a  b + 1

BÀI 5 : Cho hai tập hợp A = [a ; a + 2] và B = (5 ; 6). Tìm a để:

1) A  B 2) B  A 3) A  B = 

ĐS : 1) a   ; 2) 4  a  5 ; 3) a  3 hoặc a  6.

BÀI 6 : Cho hai tập hợp A = (2 ; 7 – m] và B = (m – 1 ; +). Xác định a để:

1) A  B 2) A  B   3) A  B = (1 ; +).

ĐS : 1) m  3 ; 2) m < 4 ; 3) m = 2.

BÀI 7 : Cho hai tập khác rỗng : A = (m – 1 ; 4] ; B = (2 ; 2m + 2) với m  R.

Xác định m trong mỗi trường hợp sau :

1) A  B   2) A  B 3) B  A 4) (A  B)  (1 ; 3)

ĐS : 1) 2 < m < 5 ; 2) 1 < m < 5 ; 3) 2 < m  1 ; 4)

2 m 1

0 

BÀI 8 : Cho hai đoạn A = [a ; a + 2), B = (1 ; 5]. Xác định a để :

1) A  B   2) AB 1 3) A  B 4) A  B  (0 ; 4]

ĐS : 1) –1 < a  5 ; 2) a = 5 ; 3) 1 < a  3 ; 4) a  2.

BÀI 9 : Tìm m sao cho : 1) (2 ; 5)  (m ; )   2) (1 ; )  (m ; )

3) (1 ; 4)  (m ; 6) = (1 ; 6)

4) (2 ; )  ( ; m) chứa đúng 3 số nguyên ĐS : 1) m < 5 ; 2) m  1 ; 3) 1  m < 4 ; 4) 1 < m  2.

BÀI 10 : Cho hai tập hợp A = (2m – 1 ; m + 3) và B = (4 ; 5), với m là số thực. Tìm m để :

1) A  B 2) B  A 3) A  B = 

ĐS : 1) m 2

2 3 

 ; 2) m   ; 3) m  7 hoặc 3  m < 4.

BÀI 11 : Cho hai tập hợp A = [–2 ; m] và B = (1 ; 5], với m là số thực. Xác định tập B \ A.

BÀI 12 : Cho hai tập hợp A = (– 3 ; 5] và B = [a ; +). Tìm a để : 1) A  B = [–2 ; 5]

2) A  B có đúng một phần tử.

ĐS : 1) a = – 2 ; 2) a = 5.

(2)

BÀI 13 : Cho hai tập hợp A = [– 4 ; 2] và B = [–8 ; a + 2]. Tìm a để A  B có vô số phần tử.

ĐS : a > –6.

BÀI 14 : Cho hai tập hợp A = [2 ; m + 1] và

 

 

 ; 2

B 1 . Tìm a để A  B chỉ có đúng một phần tử.

ĐS : m   .

BÀI 15 : Cho hai tập hợp A = (m ; m + 1) và B = (3 ; 5), với m là số thực. Tìm m để A  B là một khoảng.

ĐS : 2 < m < 5.

BÀI 16 : Cho các nửa khoảng A = (a ; a + 1] và B = [b ; b + 2).

a) Gọi C = A  B. Với điều kiện nào của a, b thì C là một đoạn. Tính độ dài của C khi đó.

b) Gọi C = A  B. Với điều kiện nào của a, b thì C là một đoạn. Tính độ dài của C khi đó.

BÀI 17 : Cho m là một tham số thực và hai tập hợp A = [1 – 2m ; m + 3] và B = x  R  x  8 – 5m.

Tìm tất cả các giá trị m để A  B = .

ĐS :

6 m 5 3 2  

BÀI 18 : Cho hai tập hợp A = x  R  mx – 3  = mx – 3 và B = x  R  x2 – 4 = 0, với m là một tham số thực. Tìm tất cả các giá trị m để B \ A = B.

ĐS :

2 m 3 2 3  

BÀI 19 : Cho A = 1 ; 2 ; 3 ; 4 ; 5 và B = 2 ; 4 ; 5 ; 6 ; 7 ; 8.

a) Xác định các tập hợp: A  B ; A  B ; A \ B ; B \ A.

b) Ký hiệu số phần tử n của một tập hợp hữu hạn A là  A  = n. Tìm  A ;  B ;  A  B ;  A  B  Từ đó suy ra công thức: AB  A  B  AB hoặc AB  A  B  AB

BÀI 20 : Mỗi học sinh của lớp 10C1 đều biết chơi đá cầu hoặc cầu lông, biết rằng có 25 em biết chơi đá cầu, 30 em biết chơi cầu lông, 15 em biết chơi cả hai. Hỏi lớp 10C1 có bao nhiêu em chỉ biết đá cầu? bao nhiêu em chỉ biết đánh cầu lông? sĩ số lớp là bao nhiêu?

ĐS : 10, 15, 40.

BÀI 21 : Trong kì thi học sinh giỏi cấp trường, lớp 10A có 17 bạn được công nhận học sinh giỏi Văn, 25 bạn học sinh giỏi Toán. Tìm số học sinh giỏi cả Văn và Toán, biết lớp 10A có 45 học sinh và có 13 học sinh không đạt học sinh giỏi.

ĐS : 10 học sinh.

BÀI 22 : Mỗi học sinh lớp 10B đều chơi bóng đá hoặc bóng chuyền. Biết rằng có 25 bạn chơi bóng đá, 20 bạn chơi bóng chuyền và 10 bạn chơi cả hai môn. Hỏi lớp 10B có bao nhiêu em chỉ chơi bóng đá? bao nhiêu em chỉ chơi bóng chuyền? sĩ số lớp 10B là bao nhiêu?

ĐS : 15, 10, 35 học sinh.

BÀI 23 : Một lớp có 45 học sinh. Mỗi em đều đăng ký chơi ít nhất một trong hai môn : bóng đá hoặc bóng chuyền. Có 35 học sinh đăng ký môn bóng đá, 15 học sinh đăng ký môn bóng chuyền. Hỏi có bao nhiêu em:

a) Đăng ký chơi cả hai môn?

b) Chỉ đăng ký chơi bóng đá?

c) Chỉ đăng ký chơi bóng chuyền?

ĐS : a) 5 ; b) 30 ; c) 10.

BÀI 24 : Trong lớp 10C2, có 15 học sinh giỏi môn Toán, 14 học sinh giỏi môn Văn và 12 học sinh giỏi môn Tiếng Anh, biết rằng có 8 học sinh vừa giỏi Văn và Toán, có 5 học sinh vừa giỏi Văn và Tiếng Anh, có 7

(3)

học sinh vừa giỏi Toán và Tiếng Anh, trong đó chỉ có đúng 11 học sinh giỏi cả hai môn. Hỏi có bao nhiêu học sinh của lớp :

a) Giỏi cả 3 môn Văn, Toán và Tiếng Anh.

b) Giỏi đúng một môn Văn, Toán hoặc Tiếng Anh.

ĐS : a) 3 ; b) 4, 3, 3.

BÀI 25 : Lớp 10B1 có 7 học sinh giỏi Toán, 5 học sinh giỏi Lý, 6 học sinh giỏi Hóa, 3 học sinh giỏi cả Toán và Lý, 4 học sinh giỏi cả Toán và Hóa, 2 học sinh giỏi cả Lý và Hóa, 1 học sinh giỏi cả 3 môn Toán, Lý, Hóa. Hỏi số học sinh giỏi ít nhất một môn (Toán, Lý, Hóa) của lớp 10B1 là bao nhiêu học sinh?

BÀI 26 : Cho A = {x  Z  x = 12, k  N}, B = {x  N  x chia hết cho 3 và 4}. Chứng minh A = B.

BÀI 27 : Gọi A, B, C, D lần lượt là tập hợp các hình bình hành, hình chữ nhật, hình thoi, hình vuông. Gọi E, G, H lần lượt là tập hợp các hình bình hành có một góc vuông, hình thoi có hai đường chéo bằng nhau. Hãy sử dụng các kí hiệu “”, “=” để thể hiện mối quan hệ giữa các tập A, B, C, D.

BÀI 28 : Cho hai tập hợp A = {3k + 1  k  N} và B = {6k + 4  k  N}. Xác định tập hợp A \ B.

BÀI 29 : Cho hai tập A, B bất kì. Chứng minh : a) Nếu B  A thì A  B = B.

b) A  B = A  B  A = B c) B  C  (A  B)  (A  C)

BÀI 30 : Cho A = {x  R x1 2}, B

m;m 3

(với m > 0).

a) Xác định m để A  B = .

b) Chứng minh rằng không tồn tại m để A  B.

BÀI 31 : Cho hai tập hợp A = {x  R  x2 < 100}, B = {x  R  x2 > 4}.

a) Các tập hợp A  Z, B  Z là hữu hạn hay vô hạn ? b) Xác định tập hợp (A  B)  N.

BÀI 32 : Cho a là số tự nhiên lớn hơn 1. Xét tập hợp khác rỗng A  N thỏa mãn điều kiện : Nếu k  A thì k + 2a  A và A

a k

 . Chứng minh rằng A = N.

BÀI 33 : Chứng minh rằng số tập hợp con của một tập hợp có n phần tử là 2n. BÀI 34 : Cho S là tập hợp gồm 51 số nguyên dương khác nhau không vượt quá 100.

Xét tập hợp T = {101 – x  x  S}.

a) Chứng minh rằng S  T  .

b) Chứng minh rằng tồn tại a, b  S sao cho a + b = 101.

BÀI 35 : Tìm tất cả các tập hợp A và B thỏa mãn đồng thời những điều kiện sau đây : (i) A  B = Z

(ii) Nếu x  A thì x – 1  B.

(iii) Nếu x, y  B thì x + y  A.

BÀI 36 : Hãy xác định các tập hợp con khác rỗng A, B, C của tập các số tự nhiên N thỏa mãn đồng thời các điều kiện sau đây :

(i) A  B = B  C = C  A =  (ii) A  B  C = N

(iii) Với mọi a  A, b  B, c  C thì a + c  A, b + c  B và a + b  C.

BÀI 37 : Cho n là số tự nhiên lớn hơn 1 và xét tập X = {1, 2, ... , 2n – 1}.

Xét tập hợp A  X thỏa mãn tính chất : (i) A có ít nhất là n – 1 phần tử

(4)

(ii) Nếu a, b  A (không nhất thiết phân biệt) thì a + b  A, miễn là a + b  X. Chứng minh tổng tất cả các phần tử của X không nằm trong A là không vượt quá n2.

BÀI 38 : Cho tập hợp X = {a ; b ; c ; d ; e ; g}. Có bao nhiêu tập con của X chứa nhiều nhất 2 phần tử ? BÀI 39 : Cho tập hợp X = {a ; b ; c ; d ; e ; g}. Có bao nhiêu tập con của X chứa đúng 3 phần tử ? BÀI 40 : Cho tập hợp X = {0 ; 1 ; 2 ; 3 ; 6}

a) Tìm hai tập con A, B của X sao cho A  B = X và tổng các phần tử của A bằng tổng các phần tử của B.

b) Tìm tất cả các tập con A của X sao cho A có đúng 3 phần tử và tổng các phần tử của A chia hết cho 3.

BÀI 41 : Cho n là số nguyên dương.

Xét tập hợp X gồm tất cả những số tự nhiên lẻ từ 1 đến 4n – 1.

a) Hỏi tập X có bao nhiêu phần tử ?

b) Chứng minh rằng tích các phần tử của tập X không vượt quá (4n2 – 1)n.

BÀI 42 : Cho S là tập hợp con của tập hợp {1, 2, ... , 1000} có tính chất : Nếu a, b là hai phần tử phân biệt của S thì a + b không thuộc S. Hỏi S có nhiều nhất là bao nhiêu phần tử ?

BÀI 43 : Cho S là tập con của tập hợp X = {1, 2, 3, ... , 100} và S 10. Chứng minh rằng tồn tại hai tập con khác rỗng X và Y của S sao cho X  Y =  và S(X) = S(Y). (S(X) kí hiệu tổng các phần tử thuộc X).

BÀI 44 : Cho X là tập hợp các số tự nhiên lớn hơn 1000. Giả sử X = A  B, A  B = . Chứng minh rằng ít nhất một trong các tập hợp A và B chứa hai phần tử khác nhau a, b sao cho a + b cũng thuộc tập hợp đó.

BÀI 45 : Gọi S là tập hợp gồm 51 số nguyên dương không vượt quá 100. Chứng minh rằng luôn tìm được hai số khác nhau a, b thuộc S sao cho a chia hết cho b.

BÀI 46 : Cho hai tập hợp A và B. Hiệu đối xứng của A và B, kí hiệu là A  B là tập hợp gồm các phần tử thuộc A hoặc B nhưng không thuộc cả A và B.

a) Chứng minh rằng nếu A  B = A thì B = .

b) Chứng minh rằng nếu A  C = B  C thì A = B.

BÀI 47 : Cho S là tập hợp gồm 100 số nguyên dương nhỏ hơn 200. Chứng minh rằng tồn tại một tập con khác rỗng T của X sao cho tích các phần tử của T là một số chính phương.

------

(5)

HƯỚNG DẪN GIẢI BÀI 1 : Có thể nói gì về các tập A và B trong mỗi trường hợp sau:

1) A  B = A. 2) A  B = B. 3) A  B = A. 4) A  B = B.

5) A \ B = A. 6) B \ A = B. 7) A \ B = B \ A. 8) X  A và X  B

 Hướng dẫn : 1) A  B = A.

Nếu A  B = A thì B là tập con của tập A, vì theo định nghĩa thì ta luôn có B  (A  B). Kiểm tra đều ngược lại cũng đúng.

Vậy A  B = A  B  A.

2) Tương tự, ta có : A  B = B  A  B.

3) A  B = A.

Nếu A  B = A thì A là tập con của tập B, vì theo định nghĩa ta luôn có (A  B)  B.

Vậy A  B = A  A  B.

4) Tương tự, ta có : A  B = B  B  A.

5) A \ B = A.

Nếu A \ B = A thì hai tập A và B phải không giao nhau.

Nếu tồn tại x  A và x  B thì do A = A \ B nên x  A \ B. Suy ra x  B (mâu thuẩn). Ngược lại, bằng cách biểu đồ Ven ta thấy nếu A  B =  thì A \ B = A cũng đúng.

Vậy A \ B = A  A  B = .

6) Tương tự, ta có : B \ A = B  A  B = .

7) A \ B = B \ A.

Nếu A \ B = B \ A thì A = B. thật vậy, nếu A  B thì phải có một phần tử của tập này nhưng không thuộc tập kia, chẳng hạn x  A và x  B suy ra x  (A \ B) nên x  (B \ A) do đó x  B và x  A (mâu thuẩn). Kiểm tra đều ngược lại cũng đúng.

Vậy A \ B = B \ A  A = B.

8) X  A và X  B

Vẽ biểu đồ Ven, ta có : X  A và X  B  X  (A  B).

Tổng quát:

1) A  B = A  B  A. 2) A  B = B  A  B.

3) A  B = A  A  B. 4) A  B = B  B  A.

5) A \ B = A  A  B = . 6) B \ A = B  A  B = .

7) A \ B = B \ A  A = B. 8) X

A B

B X

A

X   



BÀI 2 : Xét các tập hợp A = (2m – 1 ; 2m + 3) và B = (1 ; 1), với m là số thực. Tìm m để :

1) A  B 2) B  A 3) A  B = 

 Hướng dẫn :

1) Ta có A  B  1  2m – 1 < 2m + 3  1



 



 

1 m

0 m 1 3 m 2

1 m 2

1 (vô nghiệm)

Vậy không có giá trị nào của m để A  B.

2) Ta có : B  A  2m – 1  1 < 1  2m + 3



 



 

1 m

0 m 3

m 2 1

1 1 m

2  1  m  0

3) Để A  B =  thì có hai trường hợp :

TH 1 :1  2m – 1  m  1

(6)

TH 2 :2m + 3  1  m  2

Vậy A  B =  khi m  2 hoặc m  1.

 Chú ý : Bản chất của 2m – 1 là dấu “ngoặc”, còn bản chất của 1 cũng là dấu “ngoặc” nên 2m – 1 có thể bằng 1 mà không sợ nó có phần tử chung.

BÀI 3 : Cho hai đoạn A = [a ; a + 2] và B =[–1 ; 1]. Các số a, b cần thỏa mãn điều kiện gì để A  B  

 Hướng dẫn : Ta xét A  B = 

Vẽ 2 trục số hai trường hợp ra :

TH1 : a + 2 nằm bên trái b. TH2 : b + 1 nằm bên trái a.

A  B = 

 



 

3 a

1 a 1 2 a

a

1

Phủ định của mệnh đề trên là 3 a 1 3

a 1

a   



 Vậy A  B   là : – 3  a  1

BÀI 4 : Cho hai đoạn A = [a ; a + 2] và B =[b ; b + 1]. Các số a, b cần thỏa mãn điều kiện gì để AB  

 Hướng dẫn :

Vẽ 2 trục số hai trường hợp ra :

TH1 : a + 2 nằm bên trái b. TH2 : b + 1 nằm bên trái a.

Điều kiện để A  B =  là

 



1 b a

2 b a a 1 b

b 2

a

Phủ định của mệnh đề trên là b 2 a b 1 1

b a

2 b

a     



 Vậy A  B   là : b – 2  a  b + 1

BÀI 5 : Cho hai tập hợp A = [a ; a + 2] và B = (5 ; 6). Tìm a để:

1) A  B 2) B  A 3) A  B = 

 Hướng dẫn :

1) A  B  5 < a < a + 2 < 6



 



 

4 a

5 a 6 2 a

5

a  a  .

2) B  A  a  5 < 6  a + 2  a  5 và a ≥ 4  4  a  5.

3) A  B = 

 



 

6 a

3 a 6

a

5 2 a

BÀI 6 : Cho hai tập hợp A = (2 ; 7 – m] và B = (m – 1 ; +). Xác định a để:

1) A  B 2) A  B   3) A  B = (1 ; +).

 Hướng dẫn :

(7)

1) A  B  m – 1  2  m  3.

2) A  B =   7 – m  m – 1  2m ≥ 8  m ≥ 4.

Do đó A  B   là : m < 4.

 Cách khác : A  B    m – 1 < 7 – m  2m < 8  m < 4.

3) Vì A  (1 ; +) nên A  B =(1 ; +)  m – 1 = 1  m = 2.

BÀI 7 : Cho hai tập khác rỗng : A = (m – 1 ; 4] ; B = (2 ; 2m + 2) với m  R.

Xác định m trong mỗi trường hợp sau :

1) A  B   2) A  B

3) B  A 4) (A  B)  (1 ; 3)

 Hướng dẫn :

Với A = (m – 1 ; 4], B = (2 ; 2m + 2) khác tập rỗng, ta có điều kiện : 5

m 2 2

m 5 m 2

2 m 2

4 1

m   



 



 (*)

Với điều kiện (*), ta có :

1) A  B =  m 3

) ( 2 4

1 m 2 m

2  



 

lý vô Vậy A  B   là : m > –3.

So sánh kết quả này với điều kiện (*) ta thấy các giá trị m thỏa mãn yêu cầu A  B   là 2 < m < 5.

 Cách khác :

A  B    m – 1 < 2m + 2  m > 3

So sánh kết quả này với điều kiện (*) ta thấy các giá trị m thỏa mãn yêu cầu A  B   là 2 < m < 5.

2) m 1

1 m

1 m 4 2 m 2

2 1 2 m

m 2 4 1 m 2 B

A  



 



 

So sánh (*) ta thấy các giá trị m thỏa mãn yêu cầu A  B là 1 < m < 5.

3) m 1

1 m

1 m 4 2 m 2

2 1 4 m

2 m 2 2 1 m A

B  



 



 

So sánh với (*) ta thấy các giá trị m thỏa mãn yêu cầu B  A là 2 < m  1.

4) (A  B)  (1 ; 3) 



 3 2 m 2

1 1

m 

2 m 1

0  (thỏa (*)) BÀI 8 : Cho hai đoạn A = [a ; a + 2), B = (1 ; 5]. Xác định a để :

1) A  B   2) AB 1

3) A  B 4) A  B  (0 ; 4]

 Hướng dẫn :

1) A  B = 

 



 

1 a

5 a 1 2 a

5

a  A  B    –1 < a  5

2) AB 1a5

3) A  B  1 < a < a + 2  5  1 a 3

3 a

1 a 5 2 a

1

a   



 



 

4) A  B  (0 ; 4]  a + 2  4  a  2.

BÀI 9 : Tìm m sao cho :

1) (2 ; 5)  (m ; )   2) (1 ; )  (m ; )

3) (1 ; 4)  (m ; 6) = (1 ; 6) 4) (2 ; )  ( ; m) chứa đúng 3 số nguyên

(8)

 Hướng dẫn :

1) Với m  5, bằng cách biểu diễn lên trục số ta thấy (2 ; 5)  (m ; ) = .

Từ đó ta thấy rằng để (2 ; 5)  (m ; )   ta phải có m < 5.

2) m  1.

3) 1  m < 4.

4) Vì tập hợp (2 ; )  ( ; m) chứa đúng 3 số nguyên nên nó chỉ chứa các số nguyên là 1, 0 và 1.

Từ đó ta phải có 1 < m  2.

BÀI 10 : Cho hai tập hợp A = (2m – 1 ; m + 3) và B = (4 ; 5), với m là số thực. Tìm m để :

1) A  B 2) B  A 3) A  B = 

ĐS : 1) m 2

2 3 

 ; 2) m   ; 3) m  7 hoặc 3  m < 4.

 Hướng dẫn :

Điều kiện: 2m – 1 < m + 3  m < 4.

1) A  B m 2

2 3 2

m 2 m 3 5

3 m

4 1 m

2   



 



  (thỏa điều kiện)

So với điều kiện, ta có: m 2 2

3 

2) B  A



 



 

2

m 2

m 3

5 3 m

4 1 m

2 (vô nghiệm)  m  

Vậy không có giá trị nào của m thỏa mãn.

3) A  B = 

 



 

3 m

7 m 5

1 m 2

4 3

m

So với điều kiện, ta có: m  –7 hoặc (3  m < 4).

BÀI 11 : Cho hai tập hợp A = [–2 ; m] và B = (1 ; 5], với m là số thực. Xác định tập B \ A.

 Hướng dẫn :

Nếu m  1 thì A  B = . Do đó: B \ A = B.

Nếu 1 < m < 5 thì A  B = (1 ; m). Do đó: B \ A = B \ (1 ; m) = [m ; 5].

Nếu m  5 thì B  A. Do đó: B \ A = .

BÀI 12 : Cho hai tập hợp A = (– 3 ; 5] và B = [a ; +). Tìm a để :

1) A  B = [–2 ; 5] 2) A  B có đúng một phần tử.

ĐS : 1) a = – 2 ; 2) a = 5.

 Hướng dẫn :

1) A  B = [–2 ; 5] a 2

2 a

3

a  



  . Vậy a = – 2

2) A  B có đúng một phần tử  a = 5. Khi đó : A  B = 5. Vậy a – 5.

BÀI 13 : Cho hai tập hợp A = [– 4 ; 2] và B = [–8 ; a + 2]. Tìm a để A  B có vô số phần tử.

ĐS : a > –6.

 Hướng dẫn :

Điều kiện: a + 2 > –8  a > –10.

A  B có vô số phần tử khi và chỉ khi A  B có nhiều hơn một phần tử, tức là a + 2 > –4  a > –6.

Khi đó:









2 2 a khi ]

2

; 4 [ B A

2 2 a khi ] 2 a

; 4 [ B A

6 a



 

0 a khi ]

2

; 4 [ B A

0 a 6 khi ] 2 a

; 4 [ B A

Vậy a > –6 thỏa yêu cầu bài toán.

(9)

BÀI 14 : Cho hai tập hợp A = [2 ; m + 1] và

 

 

 ; 2

B 1 . Tìm a để A  B chỉ có đúng một phần tử.

ĐS : m   .

 Hướng dẫn :

Điều kiện: 2 < m + 1  m > 1.

A  B chỉ có đúng một phần tử 

2 m 1 2 1 1

m    không thỏa điều kiện  m   Vậy không có giá trị nào của m thỏa mãn.

BÀI 15 : Cho hai tập hợp A = (m ; m + 1) và B = (3 ; 5), với m là số thực. Tìm m để A  B là một khoảng.

ĐS : 2 < m < 5.

 Hướng dẫn :

Để A  B là một khoảng, ta có các trường hợp sau :

TH1 : 2 m 3

4 m 2

3 m 5

1 m 3

3

m   



 



TH2 :  



 



 m

4 m

3 m 5 1 m

3 m

TH3 : 3 m 5

4 m

5 m 3 5 1 m

5 m

3   



 



TH4 : 3 m 5

4 m

5 m 3 5 1 m

5 m

3   



 



Kết hợp các trường hợp, giá trị m cần tìm là : 2 < m < 5.

BÀI 16 : Cho các nửa khoảng A = (a ; a + 1] và B = [b ; b + 2).

a) Gọi C = A  B. Với điều kiện nào của a, b thì C là một đoạn. Tính độ dài của C khi đó.

b) Gọi C = A  B. Với điều kiện nào của a, b thì C là một đoạn. Tính độ dài của C khi đó.

 Hướng dẫn : Điều kiện: a, b  R

a) C = A  B là một đoạn  b 1 a b 2

a 1 b

2 b a b 1 a 2 b

2 b a 1 b

a 2 b a

b     



 



 

Khi đó : C = A  B = [b ; b + 2)  (a ; a + 1] = [b ; a + 1] là đoạn có độ dài a – b + 1.

b) C = A  B là một đoạn  b 1 a b

1 b a 1 b

b a 2 b 1 a b

b 2 a

b 1 a b

a    



 



 

Khi đó : C = A  B = [b ; b + 2)  (a ; a + 1] = [b ; a + 1] là đoạn có độ dài a – b + 1.

BÀI 17 : Cho m là một tham số thực và hai tập hợp A = [1 – 2m ; m + 3] và B = x  R  x  8 – 5m.

Tìm tất cả các giá trị m để A  B = .

ĐS :

6 m 5 3 2  

 Hướng dẫn :

Ta có : A = [1 – 2m ; m + 3] và B = [8 – 5m ; +).

A  B = 

6 m 5 3 2 6

m 5 3 m 2 5

m 6

2 m 3 m 5 8 3 m

3 m m 2

1   





 



 



 

BÀI 18 : Cho hai tập hợp A = x  R  mx – 3  = mx – 3 và B = x  R  x2 – 4 = 0, với m là một tham số thực. Tìm tất cả các giá trị m để B \ A = B.

(10)

ĐS :

2 m 3 2 3  

 Hướng dẫn :

Ta có : x  A  mx – 3  0.

x  B

 

2 x

2 x

Ta có : B \ A = B  A  B = 

2 m 3 2 3 0 2 m

3 0 m

2 m 3 0

m 2 3

0 m

0 m

m 2 3

0 m





















BÀI 19 : Cho A = 1 ; 2 ; 3 ; 4 ; 5 và B = 2 ; 4 ; 5 ; 6 ; 7 ; 8.

a) Xác định các tập hợp: A  B ; A  B ; A \ B ; B \ A.

b) Ký hiệu số phần tử n của một tập hợp hữu hạn A là  A  = n. Tìm  A ;  B ;  A  B ;  A  B  Từ đó suy ra công thức: AB  A  B  AB hoặc AB  A  B  AB

 Hướng dẫn :

a) Xác định các tập hợp: A  B ; A  B ; A \ B ; B \ A.

A  B = {2 ; 4 ; 5} A  B = {1 ; 2 ; 3 ; 4 ; 5 ; 6 ; 7 ; 8} A \ B = {1 ; 3} B \ A = {6 ; 7 ; 8}

b) Tìm  A ;  B ;  A  B ;  A  B  . Từ đó suy ra công thức: AB  A  B  AB A = 1 ; 2 ; 3 ; 4 ; 5   A  = 5.

B = 2 ; 4 ; 5 ; 6 ; 7 ; 8  B  = 6 A  B = {2 ; 4 ; 5}   A  B  = 3

A  B = {1 ; 2 ; 3 ; 4 ; 5 ; 6 ; 7 ; 8}   A  B  = 8 Ta thấy: 8 = 5 + 6 – 3  AB  A  B  AB

BÀI 20 : Mỗi học sinh của lớp 10C1 đều biết chơi đá cầu hoặc cầu lông, biết rằng có 25 em biết chơi đá cầu, 30 em biết chơi cầu lông, 15 em biết chơi cả hai. Hỏi lớp 10C1 có bao nhiêu em chỉ biết đá cầu? bao nhiêu em chỉ biết đánh cầu lông? sĩ số lớp là bao nhiêu?

 Hướng dẫn :

Dựa vào biểu đồ Ven ta suy ra :

Số học sinh chỉ biết đá cầu là 25 – 15 = 10.

Số học sinh chỉ biết đánh cầu lông là 30 – 15 = 15.

Do đó sĩ số học sinh của lớp 10C1 là : 10 + 15 + 15 = 40.

BÀI 21 : Trong kì thi học sinh giỏi cấp trường, lớp 10A có 17 bạn được công nhận học sinh giỏi Văn, 25 bạn học sinh giỏi Toán. Tìm số học sinh giỏi cả Văn và Toán, biết lớp 10A có 45 học sinh và có 13 học sinh không đạt học sinh giỏi.

 Hướng dẫn :

Số bạn được công nhận học sinh giỏi là: 45 – 13 = 32 (học sinh) Số học sinh giỏi cả Văn và Toán là:

10 32 25 17 B A B A B

A         (học sinh)

BÀI 22 : Mỗi học sinh lớp 10B đều chơi bóng đá hoặc bóng chuyền. Biết rằng có 25 bạn chơi bóng đá, 20 bạn chơi bóng chuyền và 10 bạn chơi cả hai môn. Hỏi lớp 10B có bao nhiêu em chỉ chơi bóng đá? bao nhiêu em chỉ chơi bóng chuyền? sĩ số lớp 10B là bao nhiêu?

Hướng dẫn :

Gọi A là tập hợp các học sinh bạn chơi bóng đá.

25 15 30

(11)

Gọi B là tập hợp các học sinh chơi bóng chuyền.

Dựa vào biểu đồ Ven ta suy ra :

Số học sinh chỉ biết chơi bóng đá là 25 – 10 = 15.

Số học sinh chỉ biết chơi bóng chuyền là 20 – 10 = 10.

Do đó sĩ số học sinh của lớp 10C1 là : 15 + 10 + 10 = 35.

 Chú ý : biết chơi bóng đá  chỉ biết chơi bóng đá. (chơi bóng đá cũng có thể chơi được bóng chuyền)

 Cách khác :

Gọi A = “Học sinh chơi bóng đá” và B = “Học sinh chơi bóng chuyền”.

A  B = “Học sinh chơi bóng đá hoặc bóng chuyền”

A  B = “Học sinh chơi cả hai môn”

Số phần tử của A  B là : 25 + 20 – 10 = 35

Số học sinh chơi bóng đá hoặc bóng chuyền là số học sinh của lớp : 35

BÀI 23 : Một lớp có 45 học sinh. Mỗi em đều đăng ký chơi ít nhất một trong hai môn : bóng đá hoặc bóng chuyền. Có 35 học sinh đăng ký môn bóng đá, 15 học sinh đăng ký môn bóng chuyền. Hỏi có bao nhiêu em:

a) Đăng ký chơi cả hai môn?

b) Chỉ đăng ký chơi bóng đá?

c) Chỉ đăng ký chơi bóng chuyền?

Chú ý: Ký hiệu số phần tử n của một tập hợp hữu hạn A là  A  = n

 Hướng dẫn :

Gọi A là tập các học sinh đăng ký chơi bóng đá.

Gọi B là tập các học sinh đăng ký chơi bóng chuyền.

Dựa vào biểu đồ Ven ta suy ra :

a) Số học sinh đăng ký chơi cả hai môn là: AB A B AB 3515455 b) Số học sinh chỉ đăng ký chơi bóng đá là: A \ AB35530

b) Số học sinh chỉ đăng ký chơi bóng chuyềnlà: B \ AB 15510.

BÀI 24 : Trong lớp 10C2, có 15 học sinh giỏi môn Toán, 14 học sinh giỏi môn Văn và 12 học sinh giỏi môn Tiếng Anh, biết rằng có 8 học sinh vừa giỏi Văn và Toán, có 5 học sinh vừa giỏi Văn và Tiếng Anh, có 7 học sinh vừa giỏi Toán và Tiếng Anh, trong đó chỉ có đúng 11 học sinh giỏi cả hai môn. Hỏi có bao nhiêu học sinh của lớp :

a) Giỏi cả 3 môn Văn, Toán và Tiếng Anh. b) Giỏi đúng một môn Văn, Toán hoặc Tiếng Anh.

 Hướng dẫn :

 Gọi V, T, A lần lượt là tập hợp các học sinh giỏi các môn Văn, Toán, Anh. Gọi C, S lần lượt là tập hợp các học sinh giỏi đúng hai môn và tập hợp các học sinh giỏi ba môn.

Gọi X là số phần tử của tập X. Dựa vào đề bài ta có biểu đồ Ven : a) Số học sinh giỏi cả ba môn Văn, Toán, Anh là :

VT

A 31

VT VA TA C

8537113

b) Số học sinh chỉ giỏi Văn là : V SVT  VA = 14 + 3 – 8 – 5 = 4 Số học sinh chỉ giỏi Toán là : T SVT  TA = 15 + 3 – 8 – 7 = 3

Số học sinh chỉ giỏi môn tiếng Anh là : A  SVA TA = 12 + 3 – 5 – 7 = 3

BÀI 25 : Lớp 10B1 có 7 học sinh giỏi Toán, 5 học sinh giỏi Lý, 6 học sinh giỏi Hóa, 3 học sinh giỏi cả Toán và Lý, 4 học sinh giỏi cả Toán và Hóa, 2 học sinh giỏi cả Lý và Hóa, 1 học sinh giỏi cả 3 môn Toán, Lý, Hóa. Hỏi số học sinh giỏi ít nhất một môn (Toán, Lý, Hóa) của lớp 10B1 là bao nhiêu học sinh?

 Hướng dẫn :

Ta dùng biểu đồ Ven để giải:

A = 25 A  B B = 15

15 10 10

A B

(12)

_ Vẽ 3 biểu đồ ven cắt nhau và chọn miền giao của 3 vòng tròn ghi số 1.

_ Vì có 3 học sinh giỏi cả Toán và Lý nên ghi số 2 vào chỗ miền chung của Toán và Lý.

_ Vì có 4 học sinh giỏi cả Toán và Hóa nên ghi số 3 vào chỗ miền chung của Toán và Hóa.

_ Vì có 2 học sinh giỏi cả Lý và Hóa nên ghi số 1 vào chỗ miền chung của Lý và Hóa.

_ Vì có 7 học sinh giỏi Toán nên ghi số 1 vào chỗ miền Toán.

_ Vì có 5 học sinh giỏi Lý nên ghi số 1 vào chỗ miền Lý.

_ Vì có 6 học sinh giỏi Hóa nên ghi số 1 vào chỗ miền Hóa.

Nhìn vào biểu đồ, số học sinh giỏi ít nhất 1 trong 3 môn là : 1 + 2 + 1 + 3 + 1 + 1 + 1 = 10.

(Giỏi ít nhất có nghĩa là giỏi 1 môn, hoặc 2 môn hoặc 3 môn đều đúng)

 Cách khác :

Số học sinh giỏi cả 3 môn là: 1 HS

Số học sinh giỏi hai môn là: (3 + 4 + 2) – 3.1 = 6 Số học sinh chỉ giỏi môn toán là:

1 1 4 3 7 C A B A

\

A        

Số học sinh chỉ giỏi môn Lý là:

1 1 2 3 5 C B B A

\

B        

Số học sinh chỉ giỏi môn Hóa là:

1 1 2 4 6 C B C A

\

C        

Số học sinh giỏi ít nhất 1 trong 3 môn là : 1 + 6 + 1 + 1 + 1 = 10.

BÀI 26 : Cho A = {x  Z  x = 12, k  N}, B = {x  N  x chia hết cho 3 và 4}. Chứng minh A = B.

 Hướng dẫn :

Với k  N, ta có : x  A  x = 12k 

 

 



 k 3 4 x

k 4 3

x  x chia hết cho 3 và 4  x  B (1) Ngược lại : x  B  x chia hết cho 3 và 4  x chia hết cho 12 (vì 3 và 4 nguyên tố cùng nhau)

 x = 12k  x  A (2)

Từ (1) và (2) ta có A = B.

BÀI 27 : Gọi A, B, C, D lần lượt là tập hợp các hình bình hành, hình chữ nhật, hình thoi, hình vuông. Gọi E, G, H lần lượt là tập hợp các hình bình hành có một góc vuông, hình thoi có hai đường chéo bằng nhau. Hãy sử dụng các kí hiệu “”, “=” để thể hiện mối quan hệ giữa các tập A, B, C, D.

 Hướng dẫn :

Vì hình vuông cũng là hình chữ nhật và hình thoi, hình chữ nhật và hình thoi cũng là hình bình hành nên : D  B  A, D  C  A.

Hình bình hành có một góc vuông là hình chữ nhật nên E = B.

Vì hình thoi có hai đường chéo bằng nhau là hình vuông, hình chữ nhật có hai cạnh liên tiếp bằng nhau cũng là hình vuông nên G = H = D.

BÀI 28 : Cho hai tập hợp A = {3k + 1  k  N} và B = {6k + 4  k  N}. Xác định tập hợp A \ B.

 Hướng dẫn :

Ta có : 3k + 1 = 6m + 4  3k = 6m + 3  k = 2m + 1

Do đó ta chia các phần tử thuộc tập A thành 2 lớp ứng với các giá trị k  N là k = 2m và k = 2m + 1.

Với k = 2m thì 3k + 1 = 6m + 1 và với k = 2m + 1 thì 3k + 2 = 6m + 4 với k, m  N.

Như vậy tập A gồm những số tự nhiên chia cho 6 dư 1 hoặc dư 4, tập B gồm những số tự nhiên chia cho 6 dư 4. Do đó B  A.

Vậy A \ B = {6k + 1  k  N}.

BÀI 29 : Cho hai tập A, B bất kì. Chứng minh : a) Nếu B  A thì A  B = B.

b) A  B = A  B  A = B

(13)

c) B  C  (A  B)  (A  C)

 Hướng dẫn :

a) Ta có (A  B)  B : theo định nghĩa.

x, x  B  x  A (vì B  A)  x  B và x  A  x  A  B Vậy : A  B = B.

b) x  A  x  A  B  x  A  B (vì A  B = A  B)  x  B Đảo lại : tương tự.

Vậy A = B.

c) Ta có : x, x  A  B  x  A và x  B (*)

Theo giả thiết B  C nên x  B  x  C Do đó (*)  x  A và x  C  x  A  C Vậy B  C  (A  B)  (A  C).

BÀI 30 : Cho A = {x  R x1 2}, B

m;m 3

(với m > 0).

a) Xác định m để A  B = .

b) Chứng minh rằng không tồn tại m để A  B.

 Hướng dẫn :

a) Ta có : x  A  x1 2  2 < x – 1 < 2  1 < x < 3 Vậy A = (1 ; 3).

Khi đó



 



 

3 m

3 m 1

m 3

1 3 B m

A

b) Ta có :



 

 m 3

1 3 m

m 3 1 m B

A (vô nghiệm)

Vậy không tồn tại m sao cho A  B.

BÀI 31 : Cho hai tập hợp A = {x  R  x2 < 100}, B = {x  R  x2 > 4}.

a) Các tập hợp A  Z, B  Z là hữu hạn hay vô hạn ? b) Xác định tập hợp (A  B)  N.

 Hướng dẫn :

a) Tập hợp A  Z gồm những số nguyên x sao cho x2 < 100  10 < x < 10 Vậy A  Z = {9 ; ... ; 9} có hữu hạn phần tử.

Ta có : 

 

 x 2 2 4 x

x2

Do đó B = ( ; 2)  (2 ; ). Từ đó B  Z = {x  Z x 2}. Tập này có vô số phần tử.

b) Tập hợp (A  B)  N gồm những số tự nhiên x sao cho 4 < x2 < 100  2 < x < 10 Vậy (A  B)  N = {3 ; 4 ; 5 ; 6 ; 7 ; 8 ; 9}.

BÀI 32 : Cho a là số tự nhiên lớn hơn 1. Xét tập hợp khác rỗng A  N thỏa mãn điều kiện : Nếu k  A thì k

+ 2a  A và A

a k

 . Chứng minh rằng A = N.

 Hướng dẫn :

Trước hết ta chứng minh 0  A.

Giả sử trái lại, gọi k > 0 là phần tử bé nhất của A. Khi đó do a > 1 nên k a k a

k  



 . Điều này vô lí vì theo giả thiết A

a k

 và k là phần tử bé nhất.

Vì 0  A nên 2a = 0 + 2a  A và 4a = 2a + 2a  A.

(14)

Suy ra A a

ka k 2

2 



 . Vậy A chứa tất cả những số chẵn.

Bây giờ giả sử n  N bất ký. Trong hai số na và na + 1 có một số chẵn.

Ta gọi số đó là 2k. Khi đó na  2k < na + a = (n + 1)a, do đó n a

k 2 

 . Vì 2k  A nên n  A. Vậy A = N.

BÀI 33 : Chứng minh rằng số tập hợp con của một tập hợp có n phần tử là 2n.

 Hướng dẫn :

Giả sử X = {a1, a2, ... , an}. Mỗi tập con A của X có thể được biểu diễn bởi dãy nhị phân (b1b2...bn) với quy ước bi = 0 nếu ai  A và bi = 1 nếu ai  A. Chẳng hạn, tập rỗng  ứng với dãy (00...0) và bản thân tập X ứng với (11...1). Như thế mỗi bi có 2 giá trị (hoặc là 0, hoặc là 1). Do đó có 2n dãy nhị phân khác nhau.

Vậy X có 2n tập con.

BÀI 34 : Cho S là tập hợp gồm 51 số nguyên dương khác nhau không vượt quá 100.

Xét tập hợp T = {101 – x  x  S}.

a) Chứng minh rằng S  T  .

b) Chứng minh rằng tồn tại a, b  S sao cho a + b = 101.

 Hướng dẫn :

a) Ta thấy nếu x  S thì 1  x  100. Do đó 1  101 – x  100.

Như thế tập hợp T có 51 phần tử là những số nguyên dương không vượt quá 100.

Nếu S  T =  thì S  T là một tập hợp gồm 51 + 51 = 102 là những số nguyên dương không vượt quá 100.

Điều này không thể xảy ra.

Vậy S  T = .

b) Theo câu a), tồn tại a  S  T. Khi đó do a  T nên có b  S sao cho a = 101 – b.

Như thế ta đã chỉ ra được a, b thuộc S mà a + b = 101.

BÀI 35 : Tìm tất cả các tập hợp A và B thỏa mãn đồng thời những điều kiện sau đây : (i) A  B = Z

(ii) Nếu x  A thì x – 1  B.

(iii) Nếu x, y  B thì x + y  A.

 Hướng dẫn :

Xét hai trường hợp sau đây :

1  A. Khi đó (ii) có 0 = 1 – 1  B. Lấy x  B. Vì 0, x  B nên theo (iii) có x = x + 0  A.

Như thế với mọi x  B ta có x  A. Như thế B  A.

Vì A  B = Z nên A = A  B = Z. Mặt khác, nếu x + 1  A thì x = x + 1 – 1  B.

Như thế B chứa tất cả các số nguyên. Vậy A = B = Z.

1  B \ A. Trước ta thấy rằng 0  B thì cũng theo (iii), 1 = 1 + 0  A, mâu thuẫn. Do đó 0  A \ B.

Từ 0  A ta có 1  B.

Ta lại có 2 = 1 + 1  A.

Nếu 2  B thì kết hợp với 1  B ta được 1  A, mâu thuẫn.

Vậy 2  A \ B. Tiếp theo nếu 3  A thì từ (ii) có 2  B, vô lí.

Bây giờ giả sử rằng 0, 2, ... , 2k – 2  A \ B và 1, 3, ... , 2k – 1  B \ A.

Xét số 2k. Nếu 2k  B thì (iii) ta được 2k – 1  A, vô lí. Vậy 2k  A \ B.

Tiếp theo xét số 2k + 1. Nếu 2k + 1  A thì 2k  B, vô lí. Vậy 2k + 1  B \ A.

Từ đó ta thấy rằng A là tập hợp các số nguyên chẵn và B là tập hợp các số nguyên lẻ.

BÀI 36 : Hãy xác định các tập hợp con khác rỗng A, B, C của tập các số tự nhiên N thỏa mãn đồng thời các điều kiện sau đây :

(15)

(i) A  B = B  C = C  A =  (ii) A  B  C = N

(iii) Với mọi a  A, b  B, c  C thì a + c  A, b + c  B và a + b  C.

 Hướng dẫn :

Xét số 0. Nếu 0  A thì với b  B ta có 0 + b = b  C, mâu thuẫn. Vậy 0  A.

Tương tự 0  B, do đó 0  C.

Xét số 1. Nếu 1  C thì với a  A, b  B ta có a + k  A và b + k  B với mọi k  N.

Giả sử b > a, khi đó với k = b – a ta có a + k = a + b – a = b  A, mâu thuẫn. Vậy 1  C.

Thành thử ta phải có 1  A hoặc 1  B.

a) Xét 1  A. Xét số 2. Nếu 2  C thì 1 + 2 = 3  A, suy ra 3 + 2 = 5  A.

Một cách tổng quát 2k + 1  A với mọi k  N.

Xét số b  B. Do A  B =  nên b = 2k, suy ra 2k + 1  C, mâu thuẫn với điều kiện A  C = .

Vậy 2  C.

Nếu 2  A thì với b  B ta có b + 1  C và b + 2  C. Nhưng vì b + 1  C và 1  A nên b + 2  A, mâu thuẫn. Thành thử 2  A. Suy ra 2  B. Tóm lại, ta có 0  C, 1  A và 2  B.

Từ đó bằng quy nạp ta được : A  {3k + 1, k  N}, B  {3k + 2, k  N}, C  {3k, k  N}.

Chú ý rằng mọi số tự nhiên n đều có biểu diễn duy nhất n = 3k + r với r  {0, 1, 2} nên : A = {3k + 1, k  N}, B = {3k + 2, k  N}, C = {3k, k  N}

b) Xét 1  B. Lập luận như trên ta được :

A = {3k + 2, k  N}, B = {3k + 1, k  N}, C = {3k, k  N}.

BÀI 37 : Cho n là số tự nhiên lớn hơn 1 và xét tập X = {1, 2, ... , 2n – 1}.

Xét tập hợp A  X thỏa mãn tính chất : (i) A có ít nhất là n – 1 phần tử

(ii) Nếu a, b  A (không nhất thiết phân biệt) thì a + b  A, miễn là a + b  X. Chứng minh tổng tất cả các phần tử của X không nằm trong A là không vượt quá n2.

 Hướng dẫn :

Giả sử A có m phần tử là 1  a1 < a2 < ... < am  2n – 1 với m  n – 1.

Trước hết ta sẽ chứng minh rằng với mỗi i (1  i  m) ta có ai + am – i + 1  2n.

Thật vậy, nếu trái lại thì : ai + a1 < ai + a2 < ... < ai + am – i < ai + am – i + 1

Theo (ii), tất cả m – i + 1 các số trên đều nằm trong A và các số này đều lớn hơn ai. Điều này vô lý vì chỉ có m – i số của A lớn hơn ai.

Tổng tất cả các số thuộc A là :

     

n mn n

n 1

2 1 a a 2

a a a

a a a a

a

a m

1 i m

1 i

i m i 1

m 1

m 2 m m 1

2

1      

 

 

 

Do đó tổng S của tất cả các số của X mà không nằm trong A là :

   

2

m 1

i i

n 1 i

n 1 n n 1 n 2 n a i

S

    

 Chú ý : Có thể chỉ ra tập A sao cho S = n2 là A = {2, 4, 6, ... , 2n – 2}.

BÀI 38 : Cho tập hợp X = {a ; b ; c ; d ; e ; g}.

Có bao nhiêu tập con của X chứa nhiều nhất 2 phần tử ?

 Hướng dẫn :

Tập con của X không chứa phần tử nào là tập .

Vì tập X có 6 phần tử nên có 6 tập con của X chứa 1 phần tử.

Các tập con của X có 2 phần tử được thành lập như sau :

Giả sử tập con cần tìm có dạng {x, y}. Vì X có 6 phần tử nên có 6 cách chọn phần tử x.

Sau khi chọn phần tử x thì X chỉ còn 5 phần tử, do đó với mỗi cách chọn phần tử x, ta có 5 cách chọn phần tử y, như vậy có 6.5 = 30 cặp phần tử (x ; y).

(16)

Nhưng với cách chọn trên thì với hai phần tử bất kì x, y tuần tự ta đã chọn lặp lại 2 lần là (x ; y) và (y ; x) nhưng chỉ có 1 tập duy nhất {x, y}.

Như vậy, ta đã có số lần chọn gấp đôi số tập con gồm có hai phần tử, vậy có 15 2

30  tập con gồm hai phần tử. Vậy tập con của X chứa nhiều nhất 2 phần tử là 6 + 15 + 1 = 22 tập.

BÀI 39 : Cho tập hợp X = {a ; b ; c ; d ; e ; g}.

Có bao nhiêu tập con của X chứa đúng 3 phần tử ?

 Hướng dẫn :

Giả sử tập con của X cần tìm có dạng {m ; n ; k}. Vì X có 6 phần tử nên có 6 cách chọn phần tử m. Sau khi chọn phần tử m thì X chỉ còn 5 phần tử, do đó với mỗi cách chọn phần tử m, ta có 5 cách chọn phần tử n.

Sau khi chọn các phần tử m, n thì X chỉ còn 4 phần tử, do đó có 4 cách chọn phần tử k.

Do đó có 6.5.4 = 120 bộ gồm 3 phần tử (m ; n ; k).

Nhưng với cách chọn trên thì với ba phần tử bất kì m, n, k theo thứ tự như trên ta đã chọn lặp lại 6 lần là : (m ; n ; k), (m ; k ; n), (n ; m ; k), (n ; k ; m), (k ; n ; m), (k ; m ; n) nhưng chỉ có 1 tập duy nhất {m ; n ; k}.

Như vậy, số lần chọn trên gấp sáu lần số tập con gồm có ba phần tử.

Vậy có 20

6

120  tập con gồm ba phần tử.

BÀI 40 : Cho tập hợp X = {0 ; 1 ; 2 ; 3 ; 6}

a) Tìm hai tập con A, B của X sao cho A  B = X và tổng các phần tử của A bằng tổng các phần tử của B.

b) Tìm tất cả các tập con A của X sao cho A có đúng 3 phần tử và tổng các phần tử của A chia hết cho 3.

 Hướng dẫn :

a) Ta thấy tổng các phần tử của X là 0 + 1 + 2 + 3 + 6 = 12 nên tồn tại hai tập A và B thỏa mãn đầu bài thì tổng số các phần tử trong mỗi tập phải bằng 6.

Từ đó ta có kết quả là : A = {6}, B = {0, 1, 2, 3} hoặc A = {0, 6}, B = {1, 2, 3}

b) Ta thấy trong các phần tử của A thì 0, 3, 6 đều chia hết cho 3. Hai phần tử 1 và 2 không chia hết cho 3 nhưng 1 + 2 chia hết cho 3. Từ đó các tập A thỏa mãn đầu bài chia làm 2 loại :

Loại không chứa 1 và 2 : chỉ có duy nhất tập {0 ; 3 ; 6}

Loại chứa cả 1 và 2 : có các tập {0 ; 1 ; 2}, {1 ; 2 ; 3}, {1 ; 2 ; 6}

Vậy có 4 tập hợp thỏa mãn yêu cầu bài toán.

BÀI 41 : Cho n là số nguyên dương.

Xét tập hợp X gồm tất cả những số tự nhiên lẻ từ 1 đến 4n – 1.

a) Hỏi tập X có bao nhiêu phần tử ?

b) Chứng minh rằng tích các phần tử của tập X không vượt quá (4n2 – 1)n.

 Hướng dẫn :

a) Số các phần tử của X là số số lẻ từ 1 đến 4n – 1. Như vậy X có 2n phần tử.

b) Ta thấy X có 2n phần tử, trong đó có n phần tử nhỏ hơn 2n và n phần tử lớn hơn 2n.

Những phần tử của X nhỏ hơn 2n có dạng 2n – k và những phần tử của X lớn hơn 2n có dạng 2n + k, ở đây 1

 k  2n – 1. Như thế tất cả 2n phần tử của X được chia thành n cặp (2n – k, 2n + k).

Tích của mỗi cặp như thế là : (2n – k)(2n + k) = 4n2 – k2  4n2 – 1 Suy ra tích tất cả các phần tử của X không vượt quá (4n2 – 1)n.

BÀI 42 : Cho S là tập hợp con của tập hợp {1, 2, ... , 1000} có tính chất : Nếu a, b là hai phần tử phân biệt của S thì a + b không thuộc S. Hỏi S có nhiều nhất là bao nhiêu phần tử ?

 Hướng dẫn :

Trước hết ta thấy rằng vơi S = {500, 501, ... , 1000} thì S có tính chất đòi hỏi.

Ta sẽ chứng minh rằng mọi tập S có tính chất như đầu bài chứa nhiều nhất là 501 phần tử.

Vì S có hữu hạn phần tử nên ta gọi a là phần tử nhỏ nhất của X.

Xét hai tập hợp : X = {x  S  x  a}, Y = {x + a  x  S}. Rõ ràng X và Y không giao nhau (X  Y = ) và X  Y và X, Y là tập con của tập T = {a + 1, a + 2, ... , a + 1000}.

(17)

Mà T 1000 nên X Y  T 1000, suy ra X 500. Từ đó S  X1501.

BÀI 43 : Cho S là tập con của tập hợp X = {1, 2, 3, ... , 100} và S 10. Chứng minh rằng tồn tại hai tập con khác rỗng X và Y của S sao cho X  Y =  và S(X) = S(Y). (S(X) kí hiệu tổng các phần tử thuộc X).

 Hướng dẫn :

Số tập con khác rỗng của S là : 210 – 1 = 1023

Với mỗi tập con khác rỗng A của S, ta tính tổng S(A) tất cả các phần tử của A.

Khi đó ta có 1023 tổng S(A) như thế và 1  S(A)  100 + 99 + ... + 92 + 91 < 1000.

Theo nguyên lý Dirichlet, tồn tại hai tập con A, B của S sao cho S(A) = S(B).

Bây giờ, bằng cách chọn X = A \ (A  B) và Y = B \ (A  B) ta được hai tập X, Y cần tìm.

BÀI 44 : Cho X là tập hợp các số tự nhiên lớn hơn 1000. Giả sử X = A  B, A  B = . Chứng minh rằng ít nhất một trong các tập hợp A và B chứa hai phần tử khác nhau a, b sao cho a + b cũng thuộc tập hợp đó.

 Hướng dẫn :

Do X là tập vô hạn nên một trong hai tập A hoặc B là vô hạn. Ta giả sử A vô hạn.

Lấy x, y  A. Do A vô hạn nên tồn tại z  A với z > 2x + y + 1000.

Nếu x + y  A hoặc y + z  A thì ta có điều phải chứng minh.

Giả sử trái lại là cả x + y và y + z đều không thuộc A. Như thế x + y  B, y + z  B.

Xét số z – x. Ta có : z – x > 1000, z – x > x, z – x > x + y

z – x  A. Ta có điều phải chứng minh bằng cách chọn a = x, b = z – x.

z – x  B. Bằng cách chọn a = z – x, b = x + y ta có a + b = y + z  B và có điều phải chứng minh.

BÀI 45 : Gọi S là tập hợp gồm 51 số nguyên dương không vượt quá 100. Chứng minh rằng luôn tìm được hai số khác nhau a, b thuộc S sao cho a chia hết cho b.

 Hướng dẫn :

Mỗi số x thuộc S luôn viết được duy nhất dưới dạng x = 2k.m, ở đây k là số tự nhiên, m < 100 là lẻ.

Ta thấy chỉ có 50 số lẻ nhỏ hơn 100 mà S có 51 phần tử nên tồn tại hai số x, y thuộc S sao cho x = 2k.m và y

= 2n.m với k  n. Nếu k > n thì x chia hết cho y, nếu k < n thì y chia hết cho x.

BÀI 46 : Cho hai tập hợp A và B. Hiệu đối xứng của A và B, kí hiệu là A  B là tập hợp gồm các phần tử thuộc A hoặc B nhưng không thuộc cả A và B.

a) Chứng minh rằng nếu A  B = A thì B = .

b) Chứng minh rằng nếu A  C = B  C thì A = B.

 Hướng dẫn :

a) Ta chứng minh bằng phản chứng.

Giả sử trái lại B  . Khi đó tồn tại x  B.

Ta xét quan hệ giữa phần tử x và tập A.

Nếu x  A thì do x  B nên x  A  B = A. Vô lí.

Nếu x  A thì x  B \ A. Từ đó x  A  B, nghĩa là x  A, cũng vô lí. Do đó B = .

b) Ta cũng chứng minh bằng phản chứng.

Nếu A  B. Khi đó tồn tại một phần tử thuộc tập này mà không thuộc tập kia.

Ta giả sử có x  A nhưng x  B (trường hợp còn lại tương tự).

Nếu x  C thì x  C \ B và như thế x  B  C. Từ giả thiết suy ra x  A  C.

Điều này mâu thuẫn vì từ x  A và x  C ta suy ra x  A  C.

Nếu x  C thì ta có x  A  C và x  B  C. Từ giả thiết có ngay điều vô lí.

(18)

BÀI 47 : Cho S là tập hợp gồm 100 số nguyên dương nhỏ hơn 200. Chứng minh rằng tồn tại một tập con khác rỗng T của X sao cho tích các phần tử của T là một số chính phương.

 Hướng dẫn :

Số các số nguyên tố bé hơn 200 là Ta, kí hiệu p(A) là tích tất cả các phần tử của A.

Với mỗi tập con A của X, ta viết p(A) = k2.mA, ở đây k2 là ước chính phương lớn nhất của số p(A) và mA là phần không chính phương của p(A).

Chú ý rằng mA là tích của các thừa số nguyên tố phân biệt nhỏ hơn 200.

Do đó có nhiều nhất là 299 giá trị cho mA.

Mặt khác, số các tập con khác rỗng của S là 2100 – 1 > 299.

Từ đó theo nguyên lý Dirichlet, tồn tại hai tập con A và B của S sao cho mA = mB. Bây giờ, giả sử p(A) = k2.mA, p(B) = n2.mB. Khi đó : p(A).p(B) = k2n2.(mA)2

Xét T = A  B. Do A  B nên T  . Mỗi số trong A  B được tính hai lần trong tích p(A).p(B) Từ đó ta có p(A).p(B) = p(T).(p(A  B))2  p(T) là số chính phương.

------

Tài liệu tham khảo

Tài liệu liên quan

Bôûi vì neáu phöông trình löôïng giaùc coù ñieàu kieän, khi ta giaûi ra nghieäm maø khoâng kieåm tra hoï nghieäm ñoù coù thoûa maõn hay khoâng, hay chæ thoûa

a) Ñoà thò haøm soá lieân tuïc treân moät khoaûng laø ñöôøng lieân tuïc treân khoaûng ñoù. a) Toång, hieäu, tích, thöông cuûa hai haøm soá lieân tuïc taïi moät

 Ñònh lyù : Moïi haøm soá f(x) lieân tuïc treân ñoaïn [a ; b] ñeàu coù nguyeân haøm treân ñoaïn ñoù.. 2) Xeùt xem haøm soá döôùi daáu tích phaân coù bieåu thöùc naøo

 Ñònh lyù : Moïi haøm soá f(x) lieân tuïc treân ñoaïn [a ; b] ñeàu coù nguyeân haøm treân ñoaïn ñoù.. 2) Xeùt xem haøm soá döôùi daáu tích phaân coù bieåu thöùc naøo

Goïi I laø taâm cuûa hình vuoâng A’B’C’D’ vaø M laø ñieåm thuoäc ñoaïn thaúng OI sao cho MO = 2MI (tham khaûo hình veõ)A. Khi ñoù coâsin cuûa goùc taïo bôûi

 Böôùc 2 : Töø ñieàu giaû ñònh ñoù (töùc A sai) ta suy ra ñöôïc moät ñieàu traùi vôùi moät tính chaát ñaõ ñöôïc thöøa nhaän, moät giaû thieát hoaëc moät ñònh

Trong khoaûng thôøi gian 1 giôø keå töø khi baét ñaàu chuyeån ñoäng, ñoà thò ñoù laø moät phaàn cuûa ñöôøng parabol coù ñænh I(2 ; 9) vaø truïc ñoái xöùng song

vieát phöông trình maët phaúng (P) ñi qua hai ñieåm A, M vaø song song vôùi BC 1. Tính ñoä daøi ñoaïn MN. Goïi M vaø N laàn löôït laø trung ñieåm AB vaø CD. 1)